LSAT and Law School Admissions Forum

Get expert LSAT preparation and law school admissions advice from PowerScore Test Preparation.

 rachue
  • Posts: 140
  • Joined: Jun 22, 2011
|
#1630
Hi there,

I just wanted to verify that I understand completely why C was correct and B and A had to be wrong.

Is B wrong because it doesn't refer to Parent P's proposal of computers being introduced to children and computer languages taught in high school?

Also, is A wrong because it refers to "regular training" which wasn't mentioned in the stimulus and could refer to adults having regular training rather than children and hs students?

And then I'm guessing that C is correct because it refers to Parent P's proposal while also attacking Parent Q's conclusion that it would be pointless to teach computer skills?


Is all of this valid or am I missing some things? Thanks in advance.
 Nikki Siclunov
PowerScore Staff
  • PowerScore Staff
  • Posts: 1362
  • Joined: Aug 02, 2011
|
#1632
Answer choice (A) is incorrect, because parent P never suggested that children receive "regular training" in computers or computer languages. Her point is that computer training be introduced as early as possible, not as regularly as possible. Since answer choice (A) would not support parent P's argument, it is incorrect.

Answer choice (B) is also incorrect. Parent Q's argument is that teaching kindergartners how to use computers would simply be pointless; she never suggested that they would be incapable of adapting to new technologies. Even if answer choice (B) were true and today's children are perfectly capable of adapting to use new technologies, parent Q would still have a point arguing that introducing kindergartners to computers would be a waste of time. Since answer choice (B) does not weaken parent Q's argument, it will not provide a strong logical counter for parent P.
 rachue
  • Posts: 140
  • Joined: Jun 22, 2011
|
#1637
Thanks Nikki. That clears it up.
 est15
  • Posts: 94
  • Joined: Aug 28, 2013
|
#16101
For this question, I was stuck between B, C, and D. I thought they all helped parent P's argument. But then I read the question stem again; are two of those answers wrong because the question asks specifically for the strongest logical counter, and not just an answer that supports Parent P? What should an answer that best logically counters Parent Q have that the other incorrect answers don't have?
 Steve Stein
PowerScore Staff
  • PowerScore Staff
  • Posts: 1153
  • Joined: Apr 11, 2011
|
#16143
Hi,

That's a good question. Parent Q argues that it would be pointless to teach computer languages in kindergarten, based on the fact that current computers and languages would be obsolete by they time such children grow up.

The correct answer should be responsive to Parent Q's argument, and prove that there would be a point in teaching computer languages in kindergarten, despite the fact that technology changes so quickly. Correct answer choice (C) does so, providing a general benefit regardless of the eventual obsolescence of currently used computers and computer languages.

I hope that's helpful! Please let me know whether this is clear--thanks!

Steve
 cboles
  • Posts: 27
  • Joined: Sep 15, 2016
|
#28880
I am having trouble understanding why answer choice C is the best choice in this scenario. I chose D because the argument parent Q makes is that it is pointless to teach children anything because technology changes so quickly. I saw answer choice D as a way of saying, "Ya, but they don't have to relearn everything all over again." To me, answer choice C doesn't seem to fully address parent Q's argument.
 Claire Horan
PowerScore Staff
  • PowerScore Staff
  • Posts: 408
  • Joined: Apr 18, 2016
|
#29136
Hi Cboles,

Answer choice (C) counters Parent Q's opinion that early computer learning would be "pointless" because it includes the idea that early computer learning would make it easier for students to learn new computer languages later. Answer choice (D) is not a terrible argument, but the question stem asks for the strongest argument. In general, direct counters are stronger than arguments from analogy. We are asked to consider the answer choices as true. After all, if (C) is true, early computer learning cannot be pointless. If (D) is true, all one need do is explain how/why facility with computers is distinguishable from driving skills in this context.
User avatar
 lsatquestions
  • Posts: 66
  • Joined: Nov 08, 2021
|
#96708
Nikki Siclunov wrote: Sat Aug 27, 2011 4:44 pm Answer choice (A) is incorrect, because parent P never suggested that children receive "regular training" in computers or computer languages. Her point is that computer training be introduced as early as possible, not as regularly as possible. Since answer choice (A) would not support parent P's argument, it is incorrect.

Answer choice (B) is also incorrect. Parent Q's argument is that teaching kindergartners how to use computers would simply be pointless; she never suggested that they would be incapable of adapting to new technologies. Even if answer choice (B) were true and today's children are perfectly capable of adapting to use new technologies, parent Q would still have a point arguing that introducing kindergartners to computers would be a waste of time. Since answer choice (B) does not weaken parent Q's argument, it will not provide a strong logical counter for parent P.

Hi Nikki,

Parent P says that computers should be introduced in Kindergarten, but he/she also argues that computer languages should be taught in high school. Early introduction is just one part of the argument. Do you mind further explaining why answer A) is incorrect? Is it that we can't equate regular training with learning computer languages in high school - if so why not? Or does it have to do with the later part of the answer choice about dealing with society?
User avatar
 sunshine123
  • Posts: 44
  • Joined: Jul 18, 2022
|
#96814
Howdy!!

I understand that the credited response should be responsive to Q; it is, after all, a counter to Q. Now, my question is, whether that means the credited response will have to be responsive to the CONCLUSION of Q, or the PREMISES of Q? In general, as well, does a counter to an argument respond to the conclusion of the argument or the premises of the argument?

Thanks!
User avatar
 atierney
PowerScore Staff
  • PowerScore Staff
  • Posts: 215
  • Joined: Jul 06, 2021
|
#96840
Howdy, Howdy!

Yes, generally, it will be to the conclusion of the argument. The premises are taken as given, so you're really looking to attack the logical structure of the argument to ultimately disprove/discredit the conclusion. It sounds like you have the right idea as far as this question, so hopefully this response, if nothing else, puts your mind at ease...

But let me know, as always, if you have follow up questions.

Get the most out of your LSAT Prep Plus subscription.

Analyze and track your performance with our Testing and Analytics Package.